Welcome to PhysicsOverflow! PhysicsOverflow is an open platform for community peer review and graduate-level Physics discussion.
Please help promote PhysicsOverflow ads elsewhere if you like it.
PO is now at the Physics Department of Bielefeld University!
New printer friendly PO pages!
Migration to Bielefeld University was successful!
Please vote for this year's PhysicsOverflow ads!
Please do help out in categorising submissions. Submit a paper to PhysicsOverflow!
... see more
(propose a free ad)
With $$\Delta_F(x)=\frac1{(2\pi)^4}\int d^4ke^{-ikx}/(k^2-\mu^2+i\epsilon),$$ how can I show that the Feynman delta function satisfies the inhomogeneous [Klein-Gordon equation][1] $$(\Box +\mu^2)\Delta_F(x)=-\delta^{(4)}(x)?$$
This is problem 3.3 from Mandl & Shaw's QFT text .
[1]: http://en.wikipedia.org/wiki/Klein%E2%80%93Gordon_equation
Hi, I just saw that you tried to insert a link like it is done on SE. With our editor you can just highlight the text you want to become a link, click on the button that looks like a $\infty$ symbol which opens a popup window where you can insert the URL.
I hope you do not mind that I did with the link to the book what I assumed you wanted to achieve.
I have downvoted, not because it is homework, but because you don't show us what you have tried to far (in other words, you give me the feeling you haven't actually spend any time on the problem). For some hints, see http://physics.stackexchange.com/questions/121524/how-to-show-that-the-feynman-delta-function-satisfies-the-inhomogenous-klein-gor?noredirect=1#comment246395_121524
Compute the LHS of your 2nd equation by making use of your 1st equation.
The answer is "differentiate the expression under the integral sign". The denominator is cancelled, and you get a delta function. I don't know what the confusion is.
user contributions licensed under cc by-sa 3.0 with attribution required